Given right triangle ABC with altitude BD drawn to hypotenuse AC. If AD=40 and AC=48, what is the length of AB in simplest radical form?

Given Right Triangle ABC With Altitude BD Drawn To Hypotenuse AC. If AD=40 And AC=48, What Is The Length

Answers

Answer 1

Answer:

8[tex]\sqrt{30}[/tex]

Step-by-step explanation:

AB^2 = AD * AC

we got the answer


Related Questions

What is the average score of runa 140,96 and 13?​

Answers

[tex]\huge\textsf{Hey there!}[/tex]

[tex]\large\textsf{Formula:}[/tex]

[tex]\mathsf{\dfrac{Sum\ of\ all\ terms}{Total\ number\ of\ values\ in\ the\ data}= your\ mean/average}[/tex]

[tex]\mathsf{\dfrac{140+96+13}{3}}[/tex]

[tex]\mathsf{140+36+13}[/tex]

[tex]\mathsf{140 + 96 = \bf 236}[/tex]

[tex]\mathsf{236 + 13}[/tex]

[tex]\mathsf{ = \bf 249}[/tex]

[tex]\mathsf{\dfrac{249}{3}}[/tex]

[tex]\mathsf{= \bf 83}[/tex]

[tex]\boxed{\boxed{\large\textsf{Possible answer: \huge \bf 83}}}\huge\checkmark[/tex]

[tex]\large\textsf{Good luck on your assignment and enjoy your your day!}[/tex]

~[tex]\frak{Amphitrite1040:)}[/tex]

If f(x) = x -2 and g(x) = 2x – 6, then g(4)/f(3) =​

Answers

Answer:

Step-by-step explanation:

(2×4-6)/(3-2)=2

Answer:

[tex]{ \tt{f(x) = x - 2}} \\ { \bf{f(3) = 3 - 2 = 1}} \\ \\ { \tt{g(x) = 2x - 6}} \\ { \bf{g(4) = 2(4) - 6 = 2}} \\ \\ { \boxed{ \tt{ \frac{g(4)}{f(3)} = \frac{2}{1} = 2}}}[/tex]

Please please help me please I really need help please just tap on picture and you will see the question

Answers

Answer:

No , it is not a right angle triangle

Step-by-step explanation:

according to the pythagoras theorem in right angled triangle sum of square of two sides is equal to the square of it's hypotenuse.

using pythagoras theorem

a^2 + b^2 = c^2

9^2 + 16^2 = 25^2

81 + 256 = 625

337 = 625

since sum of square of two smallest sides of a triangle is not equal to the square of it's hypotenuse it can be concluded that the given figure does not form right angle triangle.

 Marsha has a bag that contains 4 green marbles, 8 yellow marbles , and 20 red marbles . If she chooses one marble from the bag, what is the probability that the marble is not red?

PLEASE HELP IF YOURE GOOD AT GEOMETRY!!

Answers

Answer:

C. 3/8

HOPE THIS HELPS :)

Answer:

c. 3/8

Step-by-step explanation:

first you need the denomerator by adding all marbles together which equals 32. now for the munerator you need the sum of the green and yellow marbles. this equals 12. so your fraction is 12/32. next we simplify. we can divide both numbers by 4. getting us a fraction of 3/8.

I need help I’ll give u brainlest

Answers

Answer:

[tex]V=280[/tex] cubic inches

Step-by-step explanation:

Volume formula for triangular prism is  [tex]V=\frac{1}{2} bhl[/tex]

[tex]V=\frac{1}{2} (7)(10)(8)[/tex]

[tex]V=\frac{1}{2}(560)[/tex]

[tex]V=280[/tex]

Hope this helps

Polygon ABCDE is reflected to produce polygon A’ B’ C’ D’ E’. What is the equation for the line of reflection?

Options:
A. y=0
B. x=0
C. x=2
D. y=1

Answers

Answer:

B. x=0

Step-by-step explanation:

The Polygon ABCDE is reflected by

y-axis => x=0 , to produce polygon A’ B’ C’ D’ E’

Answer:

B. x = 0

Step-by-step explanation:

Plato

he speeds (in MPH) of automobiles traveling in a city are given below:
20, 35, 42, 52, 65, 49, 24, 37, 23, 41, 50, 58
The mean speed of the cars is

Answers

Answer:

Mean speed = 41.3 mph

Step-by-step explanation:

Given that,

The speeds of an automobiles are given below:

20, 35, 42, 52, 65, 49, 24, 37, 23, 41, 50, 58

We need to find the mean speed of the cars.

Mean = sum of observations/ no. of observation

[tex]M=\dfrac{20+35+42+52+65+49+24+37+23+41+50+58}{12}\\\\M=41.3[/tex]

So, the mean speed of the cars is equal to 41.3 mph.

Determine whether the following event is mutually exclusive or not mutually exclusive.

Choosing a student who is a mathematics major or a business major from a nearby university to participate in a research study. (Assume that each student only has one major.)

Answers

Answer:

The event is mutually exclusive.

Step-by-step explanation:

Mutually exclusive events are events that cannot exist simultaneously.

Thus, events that are not mutually exclusive can exist simultaneously.

Since each student only has one major, a single student cannot be both a mathematics major and a business major.

So, the event is mutually exclusive.

Find a power series for the function, centered at c, and determine the interval of convergence. f(x) = 9 3x + 2 , c = 6

Answers

Answer:

[tex]\frac{9}{3x + 2} = 1 - \frac{1}{3}(x - \frac{7}{3}) + \frac{1}{9}(x - \frac{7}{3})^2 - \frac{1}{27}(x - \frac{7}{3})^3 ........[/tex]

The interval of convergence is:[tex](-\frac{2}{3},\frac{16}{3})[/tex]

Step-by-step explanation:

Given

[tex]f(x)= \frac{9}{3x+ 2}[/tex]

[tex]c = 6[/tex]

The geometric series centered at c is of the form:

[tex]\frac{a}{1 - (r - c)} = \sum\limits^{\infty}_{n=0}a(r - c)^n, |r - c| < 1.[/tex]

Where:

[tex]a \to[/tex] first term

[tex]r - c \to[/tex] common ratio

We have to write

[tex]f(x)= \frac{9}{3x+ 2}[/tex]

In the following form:

[tex]\frac{a}{1 - r}[/tex]

So, we have:

[tex]f(x)= \frac{9}{3x+ 2}[/tex]

Rewrite as:

[tex]f(x) = \frac{9}{3x - 18 + 18 +2}[/tex]

[tex]f(x) = \frac{9}{3x - 18 + 20}[/tex]

Factorize

[tex]f(x) = \frac{1}{\frac{1}{9}(3x + 2)}[/tex]

Open bracket

[tex]f(x) = \frac{1}{\frac{1}{3}x + \frac{2}{9}}[/tex]

Rewrite as:

[tex]f(x) = \frac{1}{1- 1 + \frac{1}{3}x + \frac{2}{9}}[/tex]

Collect like terms

[tex]f(x) = \frac{1}{1 + \frac{1}{3}x + \frac{2}{9}- 1}[/tex]

Take LCM

[tex]f(x) = \frac{1}{1 + \frac{1}{3}x + \frac{2-9}{9}}[/tex]

[tex]f(x) = \frac{1}{1 + \frac{1}{3}x - \frac{7}{9}}[/tex]

So, we have:

[tex]f(x) = \frac{1}{1 -(- \frac{1}{3}x + \frac{7}{9})}[/tex]

By comparison with: [tex]\frac{a}{1 - r}[/tex]

[tex]a = 1[/tex]

[tex]r = -\frac{1}{3}x + \frac{7}{9}[/tex]

[tex]r = -\frac{1}{3}(x - \frac{7}{3})[/tex]

At c = 6, we have:

[tex]r = -\frac{1}{3}(x - \frac{7}{3}+6-6)[/tex]

Take LCM

[tex]r = -\frac{1}{3}(x + \frac{-7+18}{3}+6-6)[/tex]

r = -\frac{1}{3}(x + \frac{11}{3}+6-6)

So, the power series becomes:

[tex]\frac{9}{3x + 2} = \sum\limits^{\infty}_{n=0}ar^n[/tex]

Substitute 1 for a

[tex]\frac{9}{3x + 2} = \sum\limits^{\infty}_{n=0}1*r^n[/tex]

[tex]\frac{9}{3x + 2} = \sum\limits^{\infty}_{n=0}r^n[/tex]

Substitute the expression for r

[tex]\frac{9}{3x + 2} = \sum\limits^{\infty}_{n=0}(-\frac{1}{3}(x - \frac{7}{3}))^n[/tex]

Expand

[tex]\frac{9}{3x + 2} = \sum\limits^{\infty}_{n=0}[(-\frac{1}{3})^n* (x - \frac{7}{3})^n][/tex]

Further expand:

[tex]\frac{9}{3x + 2} = 1 - \frac{1}{3}(x - \frac{7}{3}) + \frac{1}{9}(x - \frac{7}{3})^2 - \frac{1}{27}(x - \frac{7}{3})^3 ................[/tex]

The power series converges when:

[tex]\frac{1}{3}|x - \frac{7}{3}| < 1[/tex]

Multiply both sides by 3

[tex]|x - \frac{7}{3}| <3[/tex]

Expand the absolute inequality

[tex]-3 < x - \frac{7}{3} <3[/tex]

Solve for x

[tex]\frac{7}{3} -3 < x <3+\frac{7}{3}[/tex]

Take LCM

[tex]\frac{7-9}{3} < x <\frac{9+7}{3}[/tex]

[tex]-\frac{2}{3} < x <\frac{16}{3}[/tex]

The interval of convergence is:[tex](-\frac{2}{3},\frac{16}{3})[/tex]

The diagram shows a right-angled triangle.
xo
26 cm
17 cm
Find the size of angle x.
Give your answer correct to 1 decimal place.

Answers

Answer:

Diagram? I don't see a diagram.

Where is the diagram?

Step-by-step explanation:

if f is the function defined by f(x) = 3x-1, find the solution set for F(x)=2

Answers

Answer:

x = 1

Step-by-step explanation:

3x - 1 = 2

Add1 to both sides

3x = 3

Divide both sides by 3

x = 1

Simplify:......................................................

Answers

Answer:

...

Step-by-step explanation:...

The cut off number is 2x-1

11x+7y=17
solve for y

Answers

[tex]\implies {\blue {\boxed {\boxed {\purple {\sf {\: y = \frac{17 - 11x}{7} }}}}}}[/tex]

[tex]\large\mathfrak{{\pmb{\underline{\red{Step-by-step\:explanation}}{\red{:}}}}}[/tex]

[tex]\\11x + 7y = 17[/tex]

[tex] \\➺ \: 7y = 17 - 11x[/tex]

[tex]\\➺ \: y = \frac{17 - 11x}{7} [/tex]

[tex]\bold{ \green{ \star{ \orange{Mystique35}}}}⋆[/tex]

What is the area of this figure?

Answers

Answer:

90km² only if it is parallelogram

Step-by-step explanation:

base = 9km

height=10km

area of parallelogram = b x h

=9km x 10km

=90km²

Answer:

A = 90km2

Step-by-step explanation:

Area of a rhombus is:

1. A = s x h (if given side and height)

2. A = 1/2 a x b (if given lengths of diagonals)

3. A = s^2 sin A (if given side and length)

Therefore from your problem, height and side is given thus, you'll use number 1

A = s x h

A = 9km x 10km = 90km2

2 divided by 0.75 full divison work i dont just need the answer​

Answers

Answer:

0.375

Step-by-step explanation:

Check the picture below.

whenever we do division of decimals, we have to mind how many decimals are there on each amount, the dividend as well as the divisor, that way we pad with zeros the other amount accordingly whilst losing the dot, for example, to say divide 3 by 0.123, 3 has no decimals, whilst 0.123 has three decimals, so we can just divide 3000 by 0123, so dividing 3 by 0.123 is the same as dividing 3000 by 123.  Another example, if we were to divide say 23.761 by 555.89331, the dividend has 3 decimals, that means 3 zeros the other way, the divisor has 5 decimals, that means 5 zeros the other way while losing the dots, so we'd end up dividing 2376100000 by 55589331000, which we can simplify to just 2376100 by 5589331, as you can see in the picture in this case.

Simplify the expression.

33 · 32 + 12 ÷ 4

Answers

Answer:

1059

Step-by-step explanation:

33 · 32 + 12 ÷ 4

PEMDAS

Multiply and divide first from left to right

1056 + 3

Then add

1059

[tex]\huge\textsf{Hey there!}[/tex]

[tex]\mathsf{33\times32+12\div4}\\\\\mathsf{33\times32= \boxed{\bf 1,056}}\\\\\mathsf{\bold{1,056}+12\div4}\\\\\mathsf{12\div4=\boxed{\bf 3}}\\\\\mathsf{1,056+\bf 3}\\\mathsf{= \boxed{\bf 1,059}}\\\\\\\boxed{\boxed{\large\textsf{Answer: \huge \bf 1,059}}}\huge\checkmark[/tex]

[tex]\large\textsf{Good luck on your assignment and enjoy your day!}[/tex]

~[tex]\frak{Amphitrite40:)}[/tex]

There are 17 books on a shelf. 8 of these books are new. the rest of them are used. (GIVING BRAINLEST TO BEST ANSWER) what is the ratio?​

Answers

A. 9 to 8
B. 17 to 9?

Heyy!! Can someone help me please!!

3 (5x + 2) - 2 (4x -4)

I don’t know what to doooo!!

Answers

Answer:

7x + 14

Step-by-step explanation:

the first thing to do is expand the parentheses/brackets.

3(5x + 2) -2(4x - 4) will be

3(5x) + 3(2) -2(4x) -2(-4)

= 15x + 6 - 8x + 8

collect like terms

15x - 8x + 6 + 8 = 7x + 14

the answer is 7x + 14

Answer:

3(5x+2)-2(4x-4)

15x+6-8x+8

15x-8x+6+8

7x+14

Who can answer this? I’ll mark brainliest!!

Answers

Answer:

y = 0.48(x - 0.5)² - 3

y = 0.48(x² - x - 6)

Step-by-step explanation:

From the graph the zeros are

x = {-2, 3}

The x coordinate of the vertex is the midpoint of the roots

x = (-2 + 3) / 2

x = 0.5

The y coordinate of the vertex is

y = -3

vertex = (0.5, -3)

--------------------------------------

Merhod I - vertex

Vertex form is

y = a(x - h)² + k

plug in the vertex

y = a(x - 0.5)² - 3

to find a plug in either root

using x = 3

0 = a(3 - 0.5)² - 3

0 = a(2.5)² - 3

0 = 6.25a - 3

3 = 6.25a

a = 3/6.25

a = 0.48

y = 0.48(x - 0.5)² - 3

-----------------------------

Method II - roots

y = a(x + 2)(x - 3)

-3 = a(0.5 + 2)(0.5 - 3)

-3 = a(2.5)(-2.5)

-3 = -6.25a

3/6.25 = a

0.48 = a

y = 0.48(x + 2)(x - 3)

Expand

y = 0.48(x² - x - 6)

A person walks 1/6 mile in 1/18 hour.

The person's speed is _ miles per hour.

Answers

This Is What I Got!

Hope This Helps! :)

Have A Good Day!!

And If You Can I Wouldn't Mind A Brainliest! :))

Answer:

Divide 1/6 miles to 1/12hour since u wanna find our miles per hour

So it’ll be : 1/6 / 1/12

= 1/6 x 12/1

= 2 miles

international system of 89643092 in words​

Answers

Answer:

Eighty nine million six hundred forty three thousand ninety two

Step-by-step explanation:

89,643,092

=> Eighty nine million six hundred forty three thousand ninety two

Let V be the set of all 3x3 matrices with Real number entries, with the usual definitions of scalar multiplication and vector addition. Consider whether V is a vector space over C. Mark all true statements (there may be more than one).

a. The scalar closure axiom is satisfied
b. The additive inverse axiom is not satisfied
c The additive inverse axiom is satisfied
d. The additive closure axiom is not satisfied
e. The scalar closure axiom is not satisfied
f. The additive closure axiom is satsified
g. V is not a vector space over C
h. V is a vector space over C
i. The zero axiom is satisfied
j. The zero axiom is not satisfied

Answers

Answer:

the Scalar Closure axiom is not satisfiedV is not a Vector Space of CThe Additive Closure axiom is satisfied.

Step-by-step explanation:

According to the Question,

Given That, Let V be the set of all 3x3 matrices with Real number entries, with the usual definitions of scalar multiplication and vector addition. Consider whether V is a vector space over C.For V is a vector space over C and V is Set of 3x3 Matrices with Real entries.

Then, For any u,w ∈ V ⇒ u+w ∈ V

And u∈V and z∈C ⇒ z u ∈ V

So, If we take any z= i ∈ C

and V be any 3x3 matrices with Real entrices.

then, z,v ∉ V  ∴z,v Has Complex entries

So, the Scalar Closure axiom is not satisfied

Hence, V is not a Vector Space of C

Any u,w ∈ W ⇒ u+w ∈ V

So, The Additive Closure axiom is satisfied.

The probability distribution for a random variable x is given in the table X: -5,-3,-2,0,2,3 Probability: .17,.13,.33,.16,.11,.10 Find the probability that X <_-3

Answers

Answer:

0.3 probability that [tex]x \leq -3[/tex]

Step-by-step explanation:

The probability distribution is given in the table.

Probability that x <= -3

The values that are -3 or lower are -3 and -5. So

[tex]P(X \leq -3) = P(X = -3) + P(X = -5)[/tex]

From the table:

[tex]P(X = -3) = 0.13, P(X = -5) = 0.17[/tex]. So

[tex]P(X \leq -3) = P(X = -3) + P(X = -5) = 0.13 + 0.17 = 0.3[/tex]

0.3 probability that [tex]x \leq -3[/tex]

Answer:0.3

Step-by-step explanation:

Suppose that g(x)= f(x)+ 6. Which statement best compares the graph of g(x) with the graph of f(x)?

A. The graph of g(x) is the graph of f(x) shifted 6 units down.

B. The graph of g(x) is the graph of f(x) shifted to the right.

C. The graph of g(x) is the graph of f(x) shifted 6 units to the left.

D. The graph of g(x) is the graph of f(x) shifted 6 units up.

Answers

Answer:

D

Step-by-step explanation:

The + 6 moves it up 6 units.

The correct answer is (D) "The graph of g(x) is the graph of f(x) shifted 6 units up."

What is the function?

A relationship between a group of inputs and one output is referred to as a function. In plain English, a function is an association between inputs in which each input is connected to precisely one output. A domain, codomain, or range exists for every function. Typically, f(x), where x is the input, is used to represent a function.

When we add a constant to a function, such as in the case of g(x) = f(x) + 6, it will shift the graph of f(x) upward by 6 units.

This is because, for any value of x, the value of f(x) will be added to 6, resulting in a vertical shift of the entire graph.

Option (A) is incorrect because adding 6 to f(x) would shift the graph up, not down.

Option (B) is incorrect because adding a constant to a function does not cause it to shift horizontally.

Option (C) is incorrect because adding 6 to f(x) would shift the graph right, not left.

D. The graph of g(x) is the graph of f(x) shifted 6 units up. Adding a constant term to a function will shift the graph of the function vertically. In this case, adding 6 to f(x) will shift the graph of f(x) upward by 6 units, resulting in the graph of g(x).

Learn more about function here:

https://brainly.com/question/29633660

#SPJ7

Given points (-3;-6), G(3; -2) and H(6; 1); determine:
(a) The equation of line FG

Answers

Answer:

The equation of line FG is [tex]y = \frac{2}{3}x - 4[/tex]

Step-by-step explanation:

Equation of a line:

The equation of a line has the following format:

[tex]y = mx + b[/tex]

In which m is the slope and b is the y-intercept.

F(-3;-6), G(3; -2)

When we have two points, the slope is given by the change in y divided by the change in x. So

Change in y : -2 - (-6) = -2 + 6 = 4

Change in x: 3 - (-3) = 3 + 3 = 6

Slope: [tex]m = \frac{4}{6} = \frac{2}{3}[/tex]

So

[tex]y = \frac{2}{3}x + b[/tex]

Finding b:

(3; -2) means that when [tex]x = 3, y = -2[/tex]. We use this to find b.

[tex]y = \frac{2}{3}x + b[/tex]

[tex]-2 = \frac{2}{3}(3) + b[/tex]

[tex]2 + b = -2[/tex]

[tex]b = -4[/tex]

The equation of line FG is [tex]y = \frac{2}{3}x - 4[/tex]

Which number's estimate written as a single digit times a power of 10 will have a negative exponent?

Answers

105 i hope this helps if not then i’m sorry

A geneticist conducts an experiment with peas, one sample of offspring consisted of 450 green peas and 157 yellow peas. Based on these results, estimate the probability of getting an offspring pea that is green.

Answers

Answer: 0.738

Step-by-step explanation:

11. Mendelian Genetics. When Mendel conducted his famous genetics experiments with peas, one sample of offspring consisted of 428 green peas and 152 yellow peas. Based on those results, estimate the probability of getting an offspring pea that is green. Is the result reasonably close to the value that was expected?

p0 = 428/(428 + 152) = 0.737931

If you round your answer of 0.737931 to three decimals you will

get 0.738.

andrea is planning a birthday party and wants to include a cheese board with the desserts.
she reads online that she should have 110g of cheese per person ,but the cheese is sold in blocks of 500g
How many blocks of cheese should she buy to ensure that each guest can have 110g of cheese?​

Answers

Step-by-step explanation:

how many people in the party please ?

How many liters each of a 25% acid solution and a 50% acid solution must be used to produce 80 liters of a 40% acid solution?

Answers

Answer:

32 and 48 liters

Step-by-step explanation:

Let 25% solution is x liters, then 50% solution is (80 - x) liters.

Acid content is going to be same:

0.25x + 0.5(80 - x) = 80*0.40.25x - 0.5x + 40 = 320.25x = 8x = 8/0.25x = 32 liters

So 32 liters of 25% solution and 80 - 32 = 48 liters of 50% solution

Darcy gave her hairstylist a $ 4.90 The tip was 14​% of the cost of the haircut . Write an equation to find​ b, the cost of the haircut.

Answers

Answer:

Equation: 4.90/b = 14/100

Solution: b = $35

Step-by-step explanation:

Variable b = cost of the haircut

Solve for b:

4.90/b = 14/100

490 = 14b

35 = b

Check your work:

35 × 0.14 = 4.90

Correct!

Other Questions
Translate the description as an algebraic expression:the ratio of 3 and the product of 7 and r There may be a great deal of opposition to a later start time Choose the correct form of the adverb to complete the sentence below. Look at the degree of comparison given in parentheses to help guide you.Julie visited the old lady__ . (superlative)options:1. most often2. more often3. often The following sequence has first five terms as: -20, -17, -14, -11, -8, .......... Find an expression for the nth term of the sequence V th x hi thay i vai tr x hi khng i c th quy v cc loi c ng x hi no ? V sao ? Find a power series for the function, centered at c, and determine the interval of convergence. f(x) = 9 3x + 2 , c = 6 tan2- cot 88 evaluate which of the following is the distance between the points (3 -3) and (9 5) plz help me I'm literally about to cry Explain how to use a graph of the function f(c) to find f(3). Simplify:...................................................... On Earth, water is found naturally in all of these states EXCEPT ____A. GasB. Liquid C. SolidD. Plasma 11x+7y=17 solve for y (High points) Please solve with explanation The position vector of p (relative to the origin) is op = (x, y). if the magnitude of op is 5 units, find the set of all possible values of (x, y), with x, y E z help help help help help From what you know about Capulet, which statement best describes the reason he is so happy in this passage? In Spanish, por and para are... 1.Which of the following is a strong thesis statement?A. Inherit the Wind is a play about racism.B. Why is U2 one of the most popular bands in the world?C. Tax reform is not an adequate solution to America's current economicproblemsD. This paper will examine the causes and effects of the Civil War. The purpose of a good web page design is to make it